A museum director, in order to finance expensive new acquisitions, discreetly sold some paintings by major artists. A...

Anthony-Resendes on November 30, 2020

Help

I got this question right because I could prove 100% the other questions wrong and could not do the same for E. Is E correct because although the paintings were sold for 2-3X what they were sold for at the museum this does not necessarily prove the critics evaluation to be correct? Something else could have been the potential cause for this outcome? maybe the seller did not know how much they were worth so he tried to cheat people? Any help would be appreciated.

Reply
Create a free account to read and take part in forum discussions.

Already have an account? log in

Anna20 on January 28, 2021

I picked A - but can see now that it is incorrect because it is out of scope. I think I agree with the above, Anthony, it's a cause / effect flaw as the author concludes that the prices demonstrate the correctness of the evaluation